Đến nội dung

Hình ảnh

Tiếp sức bất đẳng thức


  • Please log in to reply
Chủ đề này có 223 trả lời

#81
ngtrungkien019a

ngtrungkien019a

    Binh nhất

  • Thành viên mới
  • 39 Bài viết

48) Giả sử $x,y,z \ge 1$ và $\sum \frac{1}{x}=1$ . Chứng minh 
$\sqrt{x+y+z} \ge \sum \sqrt{x-1}$

có sai ko bạn..cái đề ý


                     Đôi lúc bạn đối mặt với khó khăn không phải vì bạn làm điều gì đó sai mà bởi vì bạn đang đi đúng hướng.
 
 
                      
                                                           WELCOM TO My facebook


#82
luukhaiuy

luukhaiuy

    Trung sĩ

  • Thành viên
  • 176 Bài viết

48) Giả sử $x,y,z \ge 1$ và $\sum \frac{1}{x}=1$ . Chứng minh 
$\sqrt{x+y+z} \ge \sum \sqrt{x-1}$

do $\frac{1}{x}+\frac{1}{y}+\frac{1}{z}=1$ nen $(1-\frac{1}{x})+(1-\frac{1}{y})+(1-\frac{1}{z})=2$

hay $\frac{x-1}{x}+\frac{y-1}{y}+\frac{z-1}{z}=2$

Áp dụng BDT Cauchy-Schwarz ta có $\frac{x-1}{x}+\frac{y-1}{y}+\frac{z-1}{z}\geq \frac{(\sqrt{x-1}+\sqrt{y-1}+\sqrt{z-1})^2}{x+y+z}$

hay $2\geq \frac{(\sqrt{x-1}+\sqrt{y-1}+\sqrt{z-1})^2}{x+y+z}$

hay $\sqrt{2(x+y+z)}\geq \sqrt{x-1}+\sqrt{y-1}+\sqrt{z-1}$

suy ra DPCM

p/s mình nghĩ đề phải là bằng 2 chứ nhỉ nếu bằng 1 thì phải là $\sqrt{2(x+y+z)}\geq \sqrt{x-1}+\sqrt{y-1}+\sqrt{z-1}$



#83
I Love MC

I Love MC

    Đại úy

  • Thành viên nổi bật 2016
  • 1861 Bài viết

Ừ đề là $2$ nhé xin lỗi 
Bài 50:Cho $a,b,c>0$ chứng minh : 
$3(\sum a^3)^2 \ge (a^2+b^2+c^2)^3$


Bài viết đã được chỉnh sửa nội dung bởi HappyLife: 09-02-2016 - 12:54


#84
royal1534

royal1534

    Trung úy

  • Điều hành viên THCS
  • 773 Bài viết

Ừ đề là $2$ nhé xin lỗi 
50) Cho $a,b,c>0$ chứng minh : 
$3(\sum a^3)^2 \ge (a^2+b^2+c^2)^3$

Áp dụng bđt Holder cho 3 số ta có $VT=(1+1+1)(a^3+a^3+a^3)(a^3+b^3+c^3) \geq (a^2+b^2+c^2)^3$

Đẳng thức xảy ra khi $a=b=c$

 

Chứng minh bđt Holder cho bộ 3 số:

 $(a^{3}+b^{3}+c^{3})(x^{3}+y^{3}+z^{3})(m^{3}+n^{3}+p^{3}) \geq (axm+byn+czp)^{3}$

Áp dụng bất đẳng thức Cauchy ta có:

 

$\frac{a^{3}}{b^{3}+c^{3}+a^{3}}+\frac{x^{3}}{x^{3}+y^{3}+z^{3}}+\frac{m^{3}}{m^{3}+n^{3}+p^{3}} \geq \frac{3axm}{\sqrt[3]{(a^{3}+b^{3}+c^{3})(x^{3}+y^{3}+z^{3})(m^{3}+n^{3}+p^{3})}}$
Thiếp lập các bất đẳng thức tương tự và cộng lại ta có 
$\frac{a^{3}+b^{3}+c^{3}}{a^{3}+b^{3}+c^{3}}+\frac{x^{3}+y^{3}+z^{3}}{x^{3}+y^{3}+z^{3}}+\frac{m^{3}+n^{3}+p^{3}}{m^{3}+n^{3}+p^{3}}$ $\geq 3\frac{axm+byn+czp}{\sqrt[3]{(a^{3}+b^{3}+c^{3})(x^{3}+y^{3}+z^{3})(m^{3}+n^{3}+p^{3})}}$
$\rightarrow 3 \geq 3\frac{axm+byn+czp}{\sqrt[3]{(a^{3}+b^{3}+c^{3})(x^{3}+y^{3}+z^{3})(m^{3}+n^{3}+p^{3})}}$
$\rightarrow \sqrt[3]{(a^{3}+b^{3}+c^{3})(x^{3}+y^{3}+z^{3})(m^{3}+n^{3}+p^{3})} \geq axm+byn+czp$
$\rightarrow (a^{3}+b^{3}+c^{3})(x^{3}+y^{3}+z^{3})(m^{3}+n^{3}+p^{3}) \geq (axm+byn+czp)^{3} (Q.E.D)$
Dấu $'='$ xảy ra khi $a=b=c$
 


#85
I Love MC

I Love MC

    Đại úy

  • Thành viên nổi bật 2016
  • 1861 Bài viết

Bài dễ khai xuân đây : 
Bài 51. Cho $a,b,c \in \mathbb{R}$ và giả sử $a=max${$a,b,c$}. So sánh $b,c$ biết 
$(a-b)^5+(b-c)^5+(c-a)^5 \ge 0$


Bài viết đã được chỉnh sửa nội dung bởi HappyLife: 09-02-2016 - 16:54


#86
hoctrocuaHolmes

hoctrocuaHolmes

    Thượng úy

  • Thành viên
  • 1013 Bài viết

Bài 43:Cho $a,b,c$ thực dương.Chứng minh ta có BĐT sau:

 

$$\frac{a^4}{b^2}+\frac{b^4}{c^2}+\frac{c^4}{a^2}+5(ab+bc+ac)\geq 6(a^2+b^2+c^2)$$

Khai bút trên VMF =))

Đây là một bài khá khó,dạo trên AoPs thì thấy lời giải bài này :)

File gửi kèm  Problem 1.pdf   258.09K   176 Số lần tải

Một số mở rộng của bài toán này trích dẫn để mọi người tham khảo :D

  • Cho $a,b,c>0$ thì $\frac{a^4}{b^2}+\frac{b^4}{c^2}+\frac{c^4}{a^2}+6(ab+bc+ca)\geq 7(a^2+b^2+c^2)$
  • Tìm số dương $k$ lớn nhất để bất đẳng thức sau đúng:$ \frac{a^{4}}{b^{2}}+\frac{b^{4}}{c^{2}}+\frac{c^{4}}{a^{2}}\geq a^2+b^2+c^2+k[(a-b)^2+(b-c)^2+(c-a)^2] $

Chúc mừng năm mới  :D



#87
hoilamchi

hoilamchi

    Trung sĩ

  • Thành viên
  • 164 Bài viết

Bài dễ khai xuân đây : 
51) Cho $a,b,c \in \mathbb{R}$ và giả sử $a=max${$a,b,c$}. So sánh $b,c$ biết 
$(a-b)^5+(b-c)^5+(c-a)^5 \ge 0$

Mình không nghĩ bài này dễ :(

$(a-b)^5+(b-c)^5+(c-a)^5 \ge 0\Leftrightarrow (a-b)^5+(b-c)^5\geq (a-c)^5\geq 0\Rightarrow (a-b)^5+(b-c)^5\geq 0\Rightarrow (a-b)^{5}\geq (c-b)^{5}\Rightarrow \begin{bmatrix} (b-c)^{5}\geq 0 & \\ (b-c)^{5}\leq 0& \end{bmatrix}\Leftrightarrow \begin{bmatrix} a\geq b\geq c & \\ a\geq c\geq b & \end{bmatrix}$

Chắc chắn là sai rồi nhưng chẳng biết làm thế nào :(



#88
I Love MC

I Love MC

    Đại úy

  • Thành viên nổi bật 2016
  • 1861 Bài viết

Mình không nghĩ bài này dễ :(

$(a-b)^5+(b-c)^5+(c-a)^5 \ge 0\Leftrightarrow (a-b)^5+(b-c)^5\geq (a-c)^5\geq 0\Rightarrow (a-b)^5+(b-c)^5\geq 0\Rightarrow (a-b)^{5}\geq (c-b)^{5}\Rightarrow \begin{bmatrix} (b-c)^{5}\geq 0 & \\ (b-c)^{5}\leq 0& \end{bmatrix}\Leftrightarrow \begin{bmatrix} a\geq b\geq c & \\ a\geq c\geq b & \end{bmatrix}$

Chắc chắn là sai rồi nhưng chẳng biết làm thế nào :(

Ta có bổ đề sau : 
Với $x,y,z \in \mathbb{R}$ thỏa $x+y+z=0$ lúc đó $\sum x^5=2,5.xyz(\sum x^2)$



#89
Nguyenhuyen_AG

Nguyenhuyen_AG

    Trung úy

  • Thành viên nổi bật 2016
  • 945 Bài viết

Bài dễ khai xuân đây : 
51) Cho $a,b,c \in \mathbb{R}$ và giả sử $a=max${$a,b,c$}. So sánh $b,c$ biết 
$(a-b)^5+(b-c)^5+(c-a)^5 \ge 0$

 

Kết quả của bài toán được suy ra từ đẳng thức

\[(a-b)^5+(b-c)^5+(c-a)^5 = 5(a-b)(b-c)(c-a)(a^2+b^2+c^2-ab-bc-ca).\]


Nguyen Van Huyen
Ho Chi Minh City University Of Transport

#90
anhquannbk

anhquannbk

    Sĩ quan

  • Thành viên
  • 477 Bài viết

Bài 52:

Cho $ a, b, c $ các số thực không âm thỏa mãn $ ab+bc+ac=3 $

Chứng minh rằng:

$ (2a^{2}-3ab+2b^{2})(2b^{2}-3bc+2c^{2})(2c^{2}-3ac+2a^{2}) \ge \dfrac{5}{3}(a^{2}+b^{2}+c^{2})-4 $


Bài viết đã được chỉnh sửa nội dung bởi HappyLife: 09-02-2016 - 22:52


#91
Nguyenhuyen_AG

Nguyenhuyen_AG

    Trung úy

  • Thành viên nổi bật 2016
  • 945 Bài viết

Bài 52:

Cho $ a, b, c $ các số thực không âm thỏa mãn $ ab+bc+ac=3 $

Chứng minh rằng:

$ (2a^{2}-3ab+2b^{2})(2b^{2}-3bc+2c^{2})(2c^{2}-3ac+2a^{2}) \ge \dfrac{5}{3}(a^{2}+b^{2}+c^{2})-4 $

 

Ta viết bất đẳng thức lại dưới dạng thuần nhất như sau

\[\prod (2a^2-3ab+2b^2) +4\left ( \frac{ab+bc+ca}{3} \right )^3 \geqslant \frac{5}{3}(a^2+b^2+c^2)\left ( \frac{ab+bc+ca}{3} \right )^2. \quad (1)\]

Đặt $p = a+b+c,\,q=ab+bc+ca$ và $r=abc$ thì bất đẳng thức $(1)$ trở thành

\[-125r^2+2p(55q-14p^2)r+\frac{1}{27}q^2(211p^2-742q) \geqslant 0,\]

hay là

\[\frac{2}{27}\left[q^2(43p^2-121q)-8p(16p^2-45q)r\right]+\frac{125}{27}\left[p^2q^2-4q^3+2p(9q-2p^2)r-27r^2\right] \geqslant 0.\]

Chú ý rằng

\[p^2q^2-4q^3+2p(9q-2p^2)r-27r^2 = (a-b)^2(b-c)^2(c-a)^2 \geqslant 0,\]

nên ta chỉ cần chứng minh

\[q^2(43p^2-121q) \geqslant 8p(16p^2-45q)r.\]

Bất đẳng thức này đúng vì $r \leqslant \frac{q^2}{3p}$ và

\[q^2(43p^2-121q) - 8p(16p^2-45q) \cdot \frac{q^2}{3p} = \frac{1}{3}q^2(p^2-3q) \geqslant 0.\]

Bài toán được chứng minh.


Nguyen Van Huyen
Ho Chi Minh City University Of Transport

#92
kimchitwinkle

kimchitwinkle

    Thiếu úy

  • Điều hành viên THCS
  • 526 Bài viết

Bài 53: 

Cho các số thực $a,b,c$ dương thỏa mãn : $ab+bc+ca=1$ .CMR : $B=\sum \frac{a^{8}}{(b^{2}+c^{2})^{2}}\geq \frac{1}{4}$


Bài viết đã được chỉnh sửa nội dung bởi HappyLife: 09-02-2016 - 23:21


#93
kimchitwinkle

kimchitwinkle

    Thiếu úy

  • Điều hành viên THCS
  • 526 Bài viết

Bài 54: 

Cho $a,b,c,d>0$. Chứng minh rằng : $3+\frac{5}{1+a}+\frac{7}{1+a+b}+\frac{9}{1+a+b+c}+\frac{36}{1+a+b+c+d}\leq 4(1+\frac{1}{a}+\frac{1}{b}+\frac{1}{c}+\frac{1}{d})$



#94
royal1534

royal1534

    Trung úy

  • Điều hành viên THCS
  • 773 Bài viết

Bài 53: 

Cho các số thực $a,b,c$ dương thỏa mãn : $ab+bc+ca=1$ .CMR : $B=\sum \frac{a^{8}}{(b^{2}+c^{2})^{2}}\geq \frac{1}{4}$

Sử dụng bđt Cauchy-Swarchz ta có:

$3.\sum \frac{a^8}{(b^2+c^2)^2}=3\sum (\frac{a^4}{b^2+c^2})^2 \geq (\sum \frac{a^4}{b^2+c^2})^2 \geq [\frac{(a^2+b^2+c^2)^2}{2(a^2+b^2+c^2)}]^2 =\frac{1}{4}(a^2+b^2+c^2) \geq \frac{1}{4}(ab+bc+ca)=\frac{1}{4}$

$\rightarrow \sum \frac{a^8}{(b^2+c^2)^2} \geq \frac{1}{12}$

Đẳng thức xảy ra khi $a=b=c=\frac{1}{\sqrt{3}}$

P/s:Đề sai @@

-----------

Bài 55:Cho a,b,c là các số thực dương. Chứng minh: $\sum \frac{c(a^2+b^2)}{a^2(c+a)+b^2(b+c)} \leq \frac{3}{2}$ (Bài này chế :).Không biết có bị trùng với ai không )


Bài viết đã được chỉnh sửa nội dung bởi royal1534: 09-02-2016 - 23:33


#95
Gachdptrai12

Gachdptrai12

    Thượng sĩ

  • Điều hành viên THCS
  • 280 Bài viết

một bài mình tâm đắc nè 

Bài 56: cho x,y,z là các số thực c/m

$((x-y)^{3}+(y-z)^{3}+(z-x)^{3})(\frac{1}{(x-y)^{3}}+\frac{1}{(y-z)^{3}}+\frac{1}{(z-x)^{3}})\leq \frac{-45}{4}$


Bài viết đã được chỉnh sửa nội dung bởi HappyLife: 10-02-2016 - 09:45


#96
hoctrocuaHolmes

hoctrocuaHolmes

    Thượng úy

  • Thành viên
  • 1013 Bài viết

Sử dụng bđt Cauchy-Swarchz ta có:

$3.\sum \frac{a^8}{(b^2+c^2)^2}=3\sum (\frac{a^4}{b^2+c^2})^2 \geq (\sum \frac{a^4}{b^2+c^2})^2 \geq [\frac{(a^2+b^2+c^2)^2}{2(a^2+b^2+c^2)}]^2 =\frac{1}{4}(a^2+b^2+c^2) \geq \frac{1}{4}(ab+bc+ca)=\frac{1}{4}$

$\rightarrow \sum \frac{a^8}{(b^2+c^2)^2} \geq \frac{1}{12}$

Đẳng thức xảy ra khi $a=b=c=\frac{1}{\sqrt{3}}$

P/s:Đề sai @@

-----------

Bài 55:Cho a,b,c là các số thực dương. Chứng minh: $\frac{c(a^2+b^2)}{a^2(c+a)+b^2(b+c)} \leq \frac{3}{2}$ (Bài này chế :).Không biết có bị trùng với ai không )

Bài chế này có sai đề không vậy :))

$\frac{c(a^2+b^2)}{a^2(c+a)+b^2(b+c)} \leq \frac{3}{2}\Leftrightarrow 2a^2c+2b^2c\leq 3a^{3}+3b^3+3b^2c+3a^2c\Leftrightarrow 3a^{3}+3b^3+b^2c+a^2c>0\rightarrow Q.E.D$

 

Bài 47:cho x,y,z,t thoa man $\left\{\begin{matrix} x+y+z+t=0 & & \\ x^2+y^2+z^2+t^2=3 & & \end{matrix}\right.$

tim max xyzt

Bài 48:cho a là số thực dương CMR:$a^n+\frac{1}{a^n}-2\geq n^2(a+\frac{1}{a}-2)$

với n là số nguyên dương 

Bài 48:Tại đây



#97
I Love MC

I Love MC

    Đại úy

  • Thành viên nổi bật 2016
  • 1861 Bài viết

Bài 57:Dễ thôi tết mà :D 
C/m $\frac{7+2b}{1+a}+\frac{7+2c}{1+b}+\frac{7+2a}{1+c}\geq\frac{69}{4}$ với $a,b,c \in \mathbb{R^+}$ và $\sum a=1$


Bài viết đã được chỉnh sửa nội dung bởi HappyLife: 10-02-2016 - 09:44


#98
royal1534

royal1534

    Trung úy

  • Điều hành viên THCS
  • 773 Bài viết

Bài chế này có sai đề không vậy :))

$\frac{c(a^2+b^2)}{a^2(c+a)+b^2(b+c)} \leq \frac{3}{2}\Leftrightarrow 2a^2c+2b^2c\leq 3a^{3}+3b^3+3b^2c+3a^2c\Leftrightarrow 3a^{3}+3b^3+b^2c+a^2c>0\rightarrow Q.E.D$

 

Fixed.Thiếu kí hiệu @@



#99
Nguyenhuyen_AG

Nguyenhuyen_AG

    Trung úy

  • Thành viên nổi bật 2016
  • 945 Bài viết

một bài mình tâm đắc nè 

Bài 56: cho x,y,z là các số thực c/m

$((x-y)^{3}+(y-z)^{3}+(z-x)^{3})(\frac{1}{(x-y)^{3}}+\frac{1}{(y-z)^{3}}+\frac{1}{(z-x)^{3}})\leq \frac{-45}{4}$

 

Anh có viết một chuyên đề trong đó có nhắc đến bài toán này, em xem trong file đính kèm nhé. :)

File gửi kèm


Nguyen Van Huyen
Ho Chi Minh City University Of Transport

#100
ineX

ineX

    Sĩ quan

  • Thành viên
  • 353 Bài viết

Bài 58:Cho đa thức hệ số thực $P\left ( x \right )=x^{3}+ax^{2}+bx+c$ có ba nghiệm

Chứng minh rằng:

$12ab+27c\leq 6a^{3}+10\sqrt{(a^{2}-2b)^{3}}$

lâu lâu đổi gió chút


Bài viết đã được chỉnh sửa nội dung bởi HappyLife: 10-02-2016 - 16:49

"Tôi sinh ra là để thay đổi thế giới chứ không phải để thế giới thay đổi tôi" - Juliel

 

3cf67218ea144a6eb6caf571068071ff.1.gif





2 người đang xem chủ đề

0 thành viên, 2 khách, 0 thành viên ẩn danh